LSAT and Law School Admissions Forum

Get expert LSAT preparation and law school admissions advice from PowerScore Test Preparation.

User avatar
 Dave Killoran
PowerScore Staff
  • PowerScore Staff
  • Posts: 5852
  • Joined: Mar 25, 2011
|
#44053
Complete Question Explanation
(The complete setup for this game can be found here: lsat/viewtopic.php?t=16041)

The correct answer choice is (C)

If K is not selected then from the contrapositive of the fourth rule G cannot be selected. Thus, if F, G, and K are not selected, then J and M must be the only two tenants selected, and either J or M must be the chairperson. If J or M is chairperson, as discussed earlier Q, R, and S must also be selected. Therefore, answer choice (C) is correct.

Get the most out of your LSAT Prep Plus subscription.

Analyze and track your performance with our Testing and Analytics Package.